Eindelijk meetkunde!

Opgave - IMO 2022 dag 2 vraag 1

Zij $ABCDE$ een convexe vijfhoek met $|BC| = |DE|$. Veronderstel dat er een punt $T$
in het inwendige van $ABCDE$ is met $|TB| = |TD|, |TC| = |TE|$ en $\angle ABT = \angle TEA$. De (rechte) lijn $AB$ snijdt de (rechte) lijnen $CD$ en $CT$ respectievelijk in de punten $P$ en $Q$. Neem aan dat de punten $P, B, A, Q$ in die volgorde op de lijn liggen. De (rechte) lijn $AE$ snijdt de (rechte) lijnen $CD$ en $DT$ respectievelijk in de punten $R$ en $S$. Neem aan dat de punten $R, E, A, S$ in die volgorde op de lijn liggen. Bewijs dat de punten $P, S, Q, R$ op één cirkel liggen.

Oplossing

Bewering 1: $\triangle ETD \cong \triangle CTB$.
Bewijs: $|TB| = |TD|$, $|TC| = |TE|$, $|BC| = |DE|$.
dus $\triangle ETD \cong \triangle CTB$
QED

Laat $\angle TED = x$, $\angle TDE = y$, en $\angle TEA = z = \angle ABT$ dan ook $\angle TCB = x$ en $\angle TBC = y$. Dus $\angle AED = z+x$ en $\angle CBA = z+y$.

Dan in $\triangle SED$ hebben we $\angle ESD = 180\text{°} - x - y - z$ en in $\triangle QBC$ hebben we $\angle CQB = 180\text{°} - x - y - z$ laat dan $M = \overline{TD} \cap \overline{AB}$ en $N = \overline{TC} \cap \overline{AE}$.

Dan $$\angle NQM = \angle CQB = 180\text{°} - x - y - z = \angle ESD = \angle NSM$$
dus $M, N, Q, S$ is cyclisch.

Bewering 2: $\angle NTE = \angle MTB$.
Bewijs: Merk op dat $$\angle NTD = \angle MTC$$
dan $$\angle NTE + \angle ETD = \angle MTB + \angle BTC$$
dus, omdat $\triangle ETD \cong \triangle CTB$ $$\angle NTE = \angle MTB$$
QED

Bewering 3: $\triangle NET \sim \triangle MTB$.
Bewijs: $\angle NET = \angle AET = z = \angle ABT =\angle MBT$ en $\angle NTE = \angle MTB$
dus $\triangle NET \sim \triangle MTB$. QED

Dit geeft
\begin{equation*}
\begin{split}
&\frac{|NT|}{|TE|} = \frac{|MT|}{|TB|}\\
\iff&\frac{|NT|}{|TC|} = \frac{|MT|}{|TD|}
\end{split}
\end{equation*}

Bewering 4: $\triangle NTM \sim \triangle CTD$.
Bewijs: Volgt uit $\frac{|NT|}{|TC|} = \frac{|MT|}{|TD|} \iff \frac{|NT|}{|MT|} = \frac{|TC|}{|TD|}$ en uit $\angle CTD = \angle NTM$. QED

Dit geeft $\angle TCD = \angle TNM$ dus $NM \parallel CD$, en dus $NM \parallel PR$
dus $\angle ANM = \angle ARP$
dan $$\angle PQS = \angle MQS = \angle MNS = \angle MNA = \angle PRA =\angle PRS$$
Dus $\angle PQS = \angle PRS$
Uit dit volgt wat we wilden bewijzen wegens gelijke omtrekshoeken.